Momentum conservation at vertex in Feynman Diagrams

In summary, the conversation discusses how to conserve momentum at different vertices in Bhabha scattering diagrams. The expert explains that momentum conservation should be done separately for each vertex and that the direction of momentum used in the calculation is arbitrary. They also mention the use of delta functions in integrals to make the conservation of momentum clearer.
  • #1
Dixanadu
254
2
Hey guys,

I need help with conserving momentum at these vertices (this is Bhabha scattering):
zaYEgyB.png


So in Diagram (a), the first vertext to the left. The incoming momenta are [itex]p_{1}+p_{2}[/itex]. The outgoing momentum I'll call it [itex]p[/itex]. So...shouldnt I have [itex]p_{1}+p_{2}=p[/itex]? Furthermore, is the propagator correct if I write it like this:

[itex]\dfrac{-i\eta_{\mu\nu}}{(p_{1}+p_{2})^{2}+i\epsilon}[/itex]?

Now I don't know how to conserve momentum at the second vertex, neither do I know how to do it for any vertex in Diagram (b). It's really confusing...
 
Physics news on Phys.org
  • #2
You do momentum conservation for each vertex separately. First define in which direction your momentum is flowing (for external lines and s-channel propagators there is a natural choice, for t-channel just pick one direction) and then put momentum in equal to momentum out and then you are done. As you have noticed, this will let you get rid of some momenta, such as the s-channel propagator momentum. You might also want to rewrite things in terms of Mandelstam variables, but that is another issue.
 
  • #3
I still don't get it though. For example I have a photon propagator in each case (not sure if that's T channel or S channel) so does that have a fixed direction of momentum?

I'm still confused...I kind of see what to do but I just don't know how. Could you help me do a couple of vertices just so I get the hang of it? (also please tell me if I got the one right in my first post)
 
  • #4
Dixanadu said:
so does that have a fixed direction of momentum?
What do you mean with "direction of momentum"? The direction used in your calculation is arbitrary, but following the time axis in s-channel (like you did) is useful. A direction in space does not matter here.
 
  • #5
Hmm. I guess it would help if you told me whether I'm right in my first post about the momentum conservation...
 
  • #7
Momentum is conserved, means that if you inititally had [itex]P= p_1 + p_2 [/itex] momentum, then at the end you are having [itex] p_3 + p_4 = P [/itex] momenta.
So [itex]p_1 + p_2 = p_3 + p_4 [/itex].

When you calculate cross sections (or amplitudes) things become much clearer by having the delta function in your integrals: [itex]\delta^{(4)} (p_1 + p_2 - p_3 - p_4 ) [/itex]

If you want to insert somehow the momentum carried by the propagator [itex]q[/itex] then this delta can give you the conservation of momenta in each vertex:
[itex] \delta^{(4)} (p_1 + p_2 -q)[/itex] (or [itex]p_1+p_2 =q[/itex]) and [itex] \delta^{(4)} (q - p_3 - p_4 ) [/itex] (or [itex]q=p_3+p_4[/itex]).

As for the t-channel, you can do the same...apply a conservation of momentum delta function to each vertex, eg [itex]\delta^{(4)} (p_1 + q - p_3)[/itex] for your upper vertex. Everything is done so that you will eventually get that the initial sum of momentum= final sum of momentum, encoded in the delta function you integrate over:
[itex]\delta^{(4)} (p_1 + p_2 - p_3 - p_4 ) [/itex]
which corresponds to the [itex]1+2 \rightarrow 3+4 [/itex]
 
Last edited:

1. What is momentum conservation at vertex in Feynman Diagrams?

Momentum conservation at vertex in Feynman Diagrams is a fundamental principle in particle physics that states that the total momentum entering and exiting a vertex must be equal. This is represented by arrows on the Feynman diagram that show the direction and magnitude of the particles' momentum.

2. Why is momentum conservation important in Feynman Diagrams?

Momentum conservation is important in Feynman Diagrams because it ensures that the laws of physics, specifically the law of conservation of momentum, are upheld. It also helps to accurately depict the interactions between particles and their resulting energies.

3. How is momentum conserved at a vertex in Feynman Diagrams?

Momentum is conserved at a vertex in Feynman Diagrams by ensuring that the sum of the momenta of the incoming particles is equal to the sum of the momenta of the outgoing particles. This is represented by the arrows on the diagram that show the direction and magnitude of the particles' momenta.

4. What happens if momentum is not conserved at a vertex in Feynman Diagrams?

If momentum is not conserved at a vertex in Feynman Diagrams, it would violate the law of conservation of momentum and the diagram would not accurately represent the interaction between particles. This could also lead to incorrect calculations and predictions of particle behavior.

5. Can momentum be conserved in Feynman Diagrams even if the particles have different masses?

Yes, momentum can still be conserved in Feynman Diagrams even if the particles have different masses. This is because the momentum of a particle is defined as its mass multiplied by its velocity, so even if the masses are different, the total momentum can still be conserved if the velocities are appropriately adjusted.

Similar threads

  • High Energy, Nuclear, Particle Physics
Replies
4
Views
1K
  • High Energy, Nuclear, Particle Physics
Replies
2
Views
1K
  • High Energy, Nuclear, Particle Physics
Replies
6
Views
2K
  • High Energy, Nuclear, Particle Physics
Replies
1
Views
1K
  • High Energy, Nuclear, Particle Physics
Replies
1
Views
1K
  • High Energy, Nuclear, Particle Physics
Replies
10
Views
2K
  • High Energy, Nuclear, Particle Physics
Replies
13
Views
3K
  • Introductory Physics Homework Help
Replies
8
Views
222
  • High Energy, Nuclear, Particle Physics
Replies
13
Views
3K
  • High Energy, Nuclear, Particle Physics
Replies
1
Views
2K
Back
Top